Michael's number

Algebra Level 3

For a positive four-digit integer n , n, let T ( n ) T(n) be the number created by swapping the hundreds and thousands digits of n n and swapping the tens and units digits of n . n. There is a unique integer M M such that T ( M ) = 4 M . T(M)= 4M. What are the last three digits of M M ?

This problem is posed by Michael T .

Details and assumptions

For example, T ( 1234 ) = 2143 T(1234)=2143 and T ( 1508 ) = 5180. T(1508) = 5180.


The answer is 782.

This section requires Javascript.
You are seeing this because something didn't load right. We suggest you, (a) try refreshing the page, (b) enabling javascript if it is disabled on your browser and, finally, (c) loading the non-javascript version of this page . We're sorry about the hassle.

9 solutions

Khalid Younis
Sep 29, 2013

Let positive integers a , b , c , d < 10 a,b,c,d < 10 and a > 0 a>0

M = a b c d = 1000 a + 100 b + 10 c + d M=\overline{abcd} = 1000a+100b+10c+d

T ( M ) = 1000 b + 100 a + 10 d + c T(M) =1000b+100a+10d+c

T ( M ) = 4 M T(M)=4M

1000 b + 100 a + 10 d + c = 4000 a + 400 b + 40 c + 4 d \Rightarrow 1000b+100a+10d+c = 4000a+400b+40c+4d

600 b + 6 d = 3900 a + 39 c \Rightarrow 600b + 6d = 3900a+39c

200 b + 2 d = 1300 a + 13 c \Rightarrow 200b + 2d = 1300a +13c

2 ( 100 b + d ) = 13 ( 100 a + c ) \Rightarrow 2(100b+d) = 13(100a+c)

100 b + d = 13 ( 50 a + c 2 \Rightarrow 100b+d = 13(50a+\frac{c}{2} )

Since 100 b + d 100b+d is a 3 digit number:

50 a + c 2 < 100 50a+\frac{c}{2} <100

c < 10 c<10 hence 50 50 a + c 2 54 50 \leq 50a +\frac{c}{2} \leq 54

100 b + d 100b+d has middle digit 0 0

Checking the options, we see that 50 a + c 2 = 54 702 = 13 × 54 50a+\frac{c}{2} =54 \Leftrightarrow 702=13 \times 54

Giving a = 1 , b = 7 , c = 8 , d = 2 a=1 ,b=7 ,c=8, d=2

M = 1782 M=1782 so the last three digits are 782 \fbox{782}

Moderator note:

Nicely done!

J G
Oct 3, 2013

M M is a four-digit integer so it can be written as: M = a b c d ( 10 ) M = \overline{abcd}^{(10)}

And by definition of T ( M ) T(M) : T ( M ) = b a d c ( 10 ) T(M) = \overline{badc}^{(10)}

Now, since 4 M = T ( M ) 4M = T(M) , we have:

4 a 1 0 3 + 4 b 1 0 2 + 4 c 10 + 4 d = b 1 0 3 + a 1 0 2 + d 10 + c 4a\cdot10^3+4b\cdot10^2+4c\cdot10+4d = b\cdot10^3+a\cdot10^2+d\cdot10+c

Wich gives after simplification: 1300 a 200 b + 13 c 2 d = 0 1300a - 200b + 13c - 2d = 0

Since T ( M ) T(M) is at most a four-digit integer (can be a three-digit one if b = 0 b=0 ), it has to be lower than 10 , 000 10,000 . And since 4 M = T ( M ) 4M = T(M) , we have:

4 M < 10 , 000 M < 2 , 500 M 2 , 499 4M < 10,000 \Rightarrow M < 2,500 \Rightarrow M \leq 2,499

This means that a { 1 , 2 } a\in\{1,2\} .

However, if a = 2 a=2 , then with the possibility of having a carry out when multiplying M M by 4 4 , this gives:

b 4 a b 8 4 b 32 b\geq4a \Rightarrow b\geq8 \Rightarrow 4b\geq32

And thus: 4 M = 4 × ( a 1 0 3 + b 1 0 2 + c 10 + d ) 4 × ( a 1 0 3 + b 1 0 2 ) 4M = 4\times(a\cdot10^3+b\cdot10^2+c\cdot10+d) \geq 4\times(a\cdot10^3+b\cdot10^2)

4 M 4 a 1 0 3 + 4 b 1 0 2 8 1 0 3 + 32 1 0 2 \Rightarrow 4M \geq 4a\cdot10^3+4b\cdot10^2 \geq 8\cdot10^3+32\cdot10^2

4 M 11.2 1 0 3 > 10 , 000 > T ( M ) \Rightarrow 4M \geq 11.2\cdot10^3 > 10,000 > T(M)

We arrive at a contradiction, so a 2 a\neq 2 . Hence a = 1 a=1 .

Furthermore since there can be no carry out on the lower digit d d , we know that: c = 4 d c = 4d .

By replacing these in the previous equation, we get:

1300 200 b + 50 d = 0 130 20 b + 5 d = 0 1300 - 200b + 50d = 0 \Rightarrow 130 - 20b + 5d = 0

b = 130 + 5 d 20 \Rightarrow b = \dfrac{130 + 5d}{20}

Moreover, since c < 10 c < 10 and c = 4 d c = 4d , we have d 2 d\leq2 and thus: { c { 0 , 4 , 8 } d { 0 , 1 , 2 } \left\{\begin{array}{ccc}c &\in& \{0,4,8\} \\ d &\in& \{0,1,2\}\end{array}\right.

  • if d = 0 d=0 , then b = 130 20 N b = \frac{130}{20} \notin \mathbb{N}
  • if d = 1 d=1 , then b = 135 20 N b = \frac{135}{20} \notin \mathbb{N}
  • if d = 2 d=2 , then b = 140 20 = 7 b = \frac{140}{20} = 7

Conclusion: { a = 1 b = 7 c = 8 ( = 4 × 2 ) d = 2 \left\{\begin{array}{cccc}a &=& 1 & \\ b &=& 7 & \\ c &=& 8 & (= 4\times2) \\ d &=& 2 &\end{array}\right.

Hence: M = 1782 M = 1782 and T ( M ) = 7128 = 4 × 1782 T(M) = 7128 = 4\times1782

The answer is 782 782

You made me understand, thank you!

Patrick Doglio - 7 years, 8 months ago

simply elegant..... (y)

Anom Ahmed - 7 years, 8 months ago

Let the number be a b c d \overline{abcd} . So by condition we have, 4 a b c d = b a d c 4 * \overline{abcd}=\overline{badc}

or, equivalently on expanding, 13 ( 100 a + c ) = 2 ( 100 b + d ) 13(100a+c)=2(100b+d) .

So 13 ( 100 b + d ) 13 | (100b+d) . Testing values of 3 digit multiples of 13 where ten's digit is 0, & testing if 2 13 ( 100 b + d ) \frac{2}{13} (100b+d) also has ten's digit 0, we get a unique favorable case for a = 1 ; b = 7 ; c = 8 ; d = 2 a=1;b=7;c=8;d=2 . Thus ans is 1782 ,easily verifiable.

Yes, this works. It is not the fastest way to do it, and complete solution would be fairly painful to write (and read). Some inequalities could greatly reduce the number of cases that have to be considered.

Alexander Borisov - 7 years, 8 months ago
Ben Ferguson
Oct 2, 2013

In order to represent a 4 digit integer with variables, we use the following form:

1000a + 100b + 10c + d

Where a,b,c and d are the digits of the integer, from left to right respectively. Also, the following limitations are placed on these values:

0 <= a < 10|a is an integer 0 <= b < 10|b is an integer 0 <= c < 10|c is an integer 0 <= d < 10|d is an integer

Now, since T(M) = 1000b + 100a + 10d + c, the following is what we wish to find:

1000b + 100a + 10d + c = 4(1000a + 100b + 10c + 1d) 1000b + 100a + 10d + c = 4000a + 400b + 40c + 4d -3900a + 600b - 39c + 6d = 0 -1300a + 200b - 13c + 2d = 0 200b - 13c + 2d = 1300a

200b - 13c + 2d = 1300a

We know that a has to equal at least 1 since the number is at least 4 digits long. The maximum possible value of 200b - 13c + 2d, following the previous restrictions, is 1818. Therefore a must be 1, since any value larger than 1 would results in the right side equalling 2600, which makes the equality impossible.

If a is one, we can substitute it back into the equation as follows:

200b - 13c + 2d = 1300

Since 200b + 2d - 13c has to equal 1300, then b must equal at least 7. It it were smaller than 7, for example 6, then the equation would simplify to 2d = 13c + 100, and since the maximum limit on 2d is 18, this equation would be impossible to simplify. Any smaller inputs of b would simply make the right side bigger, making it even more impossible! Similarly, if b was larger than 7, for example 8, then the equation would be 2d + 300 = 13c. Since the maximum limit of 13c is 117, then this equation is impossible to satisfy, and any values of b higher than 7 would simply increase the left side further! Therefore, b = 7.

Substituting it back into the equation, we get the following:

2d + 100 = 13c

Since 2d + 100 will always be an even number larger than 100, c must be an even number which, when multiplied with 13, is larger than 100, but still belongs to the realm of 0 to 1, inclusive. The only value that is possible is 8, which when substituted into the equation gives us:

2d + 100 = 104 d = 2

Therefore the number M is 1783.

Mandar Sohoni
Mar 3, 2014

the units digit of M can be 1 or 2 => the tens digit of M can be 4 or 8. Let the thousands and hundreds digits of M be a and b respectively. We have 4x(1000a + 100b + 40 +1) = 1000b + 100a + 10 + 4, or, 4x(1000a + 100b + 80 + 2) = 1000b + 100a + 20 + 8. The first equation doesn't give us any viable solutions. The second one gives us a=1 and b=7, thus the number is 1782.

is there any method other than trial and error ?? And how can u say that units digit is 1 or 2 and tens digit is 4 or 8 ??

Vighnesh Raut - 7 years, 2 months ago

Log in to reply

Take the other cases, where the units digit is 3,4,5....9. Look at 3 first, if 3 is the units digit, the tens digit of M has to be 2, so M is "ab23" and T(M) is "ba32". Now the number, "ab23" times 4, will have its last digits as 92, which is not the same as T(M). Do the same for the rest, you'll find that only 1 and 2 can be the units digit of M

Mandar Sohoni - 7 years, 2 months ago

Log in to reply

got it....thnx dude

Vighnesh Raut - 7 years, 2 months ago
Ananay Agarwal
Oct 6, 2013

Let the number be a b c d \overline{abcd} . According to the question:

4 a b c d = b a d c 4\overline{abcd} = \overline{badc}

4 ( 1000 a + 100 b + 10 c + d ) = 1000 b + 100 a + 10 d + c \implies4(1000a + 100b + 10c + d) = 1000b + 100a + 10d + c

39 ( 100 a + c ) = 6 ( 100 b + d ) \implies 39(100a + c) = 6(100b + d)

39 a 0 c = 6 b 0 d 39 a 0 c / 6 = b 0 d ( 1 ) \implies 39\overline{a0c} = 6\overline{b0d} \implies 39\overline{a0c}/6 = \overline{b0d} \space\space \space (1)

Clearly, b 0 d 909 a 0 c 909 × 6 / 39 = 139.84 \overline{b0d} \leq 909 \implies \overline{a0c} \leq 909\times6/39 = 139.84

Also, 6 a 0 c 6|\overline{a0c} (from ( 1 ) (1) . The only two numbers that satisfy the criteria are 104 , 108 104, 108 . Out of these 39 × 104 / 6 = 676 39\times 104/6 = 676 , which clearly is not of the required form. 108 × 39 / 6 = 702 108\times 39/6 = 702 .

Therefore, a = 1 , b = 7 , c = 8 , d = 2 a = 1, b = 7, c = 8, d = 2 and the number equals 1782 \boxed{1782} .

Gilbert Simmons
Oct 3, 2013

We are told that if M = 1000 a + 100 b + 10 c + d M=1000a+100b+10c+d then 4000 a + 400 b + 40 c + 4 d = 1000 b + 100 a + 10 d + c 4000a+400b+40c+4d=1000b+100a+10d+c .

Therefore 1300 a 200 b + 13 c 2 d = 0 1300a-200b+13c-2d=0 . *

Considering this equation m o d 100 mod 100 , 13 c 2 d 0 13c-2d \equiv 0 .

By inspection of even values of c , c = 8 c, c=8 and d = 2 d=2 are the only possibilities.

Substituting back into * , 1300 a 200 b + 300 = 0 1300a-200b+300=0 , giving 13 a 2 b + 3 = 0 13a-2b+3=0 .

By inspection of odd values of a , a = 1 a, a=1 and b = 7 b=7 are the only feasible solutions.

Therefore M = 1782 M=1782 .

Nitin Tyagi
Oct 2, 2013

Let M=1000a+100b+10c+d where a,b,c,d are whole nos. from 0 to 9

here a,b is not = 0 (as this would lead to 3 digit no.)

T(M)=1000b+100a+10d+c

now T(M)=4M

therefore 1000b+100a+10d+c=4000a+400b+40c+4d

solving this we get

1300a+13c=200b+2d

now here a can't be 2 or above (as this would lead to b>9 which is not possible)

therefore a=1

now by hit and trial in the equation 1300+13c=200b+2d

b=7, c=8, d=2 (remember b,c,d are integers from 0-9)

therefore M=1782

last three digits of M are 782

Jan J.
Oct 1, 2013

Suppose M = 1000 a + 100 b + 10 c + d M = 1000a + 100b + 10c + d , where a , b , c , d a,b,c,d are digits, then we are given 4000 a + 400 b + 40 c + 4 d = 1000 b + 100 a + 10 d + c 4000a + 400b + 40c + 4d = 1000b + 100a + 10d + c This can be written as 3900 a 600 b + 39 c 6 d = 0 3900 a-600 b+39 c-6 d = 0 from this c c is even, hence c = 2 x c = 2x , substituting this and dividing by 6 6 we get d = 650 a 100 b + 13 x d = 650a - 100b + 13x Remember that 0 a , b , d 9 0 \leq a,b,d \leq 9 and 0 x 4 0 \leq x \leq 4 . So try x = 4 x = 4 (so that d d is still a digit). From this it follows that ( a , b , c , d ) = ( 1 , 7 , 8 , 2 ) (a,b,c,d) = (1,7,8,2) , checking we get that it is indeed a solution, we are given that the solution is unique, hence we are done, and the last three digits are 782 \boxed{782}

0 pending reports

×

Problem Loading...

Note Loading...

Set Loading...